show this inequality $ab+bc+ac+sin{(a-1)}+sin{(b-1)}+sin{(c-1)}ge 3$












17












$begingroup$



let $a,b,c>0$ and such $a+b+c=3abc$, show that
$$ab+bc+ac+sin{(a-1)}+sin{(b-1)}+sin{(c-1)}ge 3$$
Proposed by wang yong xi




since
$$dfrac{1}{ab}+dfrac{1}{bc}+dfrac{1}{ac}=3$$
so use Cauchy-Schwarz inequality we have
$$(ab+bc+ac)left(dfrac{1}{ab}+dfrac{1}{bc}+dfrac{1}{ac}right)ge 9$$
so we have
$$ab+bc+acge 3$$










share|cite|improve this question











$endgroup$












  • $begingroup$
    Maybe minimizing the function $sin(a-1)+sin(b-1)+sin(c-1)$ with the condition $a+b+c=3abc$?
    $endgroup$
    – user326159
    Jul 2 '18 at 14:15








  • 4




    $begingroup$
    Why did people vote to close this?
    $endgroup$
    – mathworker21
    Jul 5 '18 at 23:50










  • $begingroup$
    @mathworker21 They want to stop us...
    $endgroup$
    – Yuri Negometyanov
    Jul 6 '18 at 9:56






  • 2




    $begingroup$
    @mathworker21 : A good question. All three have chosen "This question is missing context or other details." .
    $endgroup$
    – user90369
    Jul 6 '18 at 9:57






  • 1




    $begingroup$
    @mishu Unfortunately this problem cannot be solved as you hoped with two seperate inequalities. The value of $sin(a-1)+sin(b-1)+sin(c-1)$ is negative at $(a,b,c)=(1/4,3/2,14)$ among other points.
    $endgroup$
    – Isaac Browne
    Jul 6 '18 at 22:26
















17












$begingroup$



let $a,b,c>0$ and such $a+b+c=3abc$, show that
$$ab+bc+ac+sin{(a-1)}+sin{(b-1)}+sin{(c-1)}ge 3$$
Proposed by wang yong xi




since
$$dfrac{1}{ab}+dfrac{1}{bc}+dfrac{1}{ac}=3$$
so use Cauchy-Schwarz inequality we have
$$(ab+bc+ac)left(dfrac{1}{ab}+dfrac{1}{bc}+dfrac{1}{ac}right)ge 9$$
so we have
$$ab+bc+acge 3$$










share|cite|improve this question











$endgroup$












  • $begingroup$
    Maybe minimizing the function $sin(a-1)+sin(b-1)+sin(c-1)$ with the condition $a+b+c=3abc$?
    $endgroup$
    – user326159
    Jul 2 '18 at 14:15








  • 4




    $begingroup$
    Why did people vote to close this?
    $endgroup$
    – mathworker21
    Jul 5 '18 at 23:50










  • $begingroup$
    @mathworker21 They want to stop us...
    $endgroup$
    – Yuri Negometyanov
    Jul 6 '18 at 9:56






  • 2




    $begingroup$
    @mathworker21 : A good question. All three have chosen "This question is missing context or other details." .
    $endgroup$
    – user90369
    Jul 6 '18 at 9:57






  • 1




    $begingroup$
    @mishu Unfortunately this problem cannot be solved as you hoped with two seperate inequalities. The value of $sin(a-1)+sin(b-1)+sin(c-1)$ is negative at $(a,b,c)=(1/4,3/2,14)$ among other points.
    $endgroup$
    – Isaac Browne
    Jul 6 '18 at 22:26














17












17








17


11



$begingroup$



let $a,b,c>0$ and such $a+b+c=3abc$, show that
$$ab+bc+ac+sin{(a-1)}+sin{(b-1)}+sin{(c-1)}ge 3$$
Proposed by wang yong xi




since
$$dfrac{1}{ab}+dfrac{1}{bc}+dfrac{1}{ac}=3$$
so use Cauchy-Schwarz inequality we have
$$(ab+bc+ac)left(dfrac{1}{ab}+dfrac{1}{bc}+dfrac{1}{ac}right)ge 9$$
so we have
$$ab+bc+acge 3$$










share|cite|improve this question











$endgroup$





let $a,b,c>0$ and such $a+b+c=3abc$, show that
$$ab+bc+ac+sin{(a-1)}+sin{(b-1)}+sin{(c-1)}ge 3$$
Proposed by wang yong xi




since
$$dfrac{1}{ab}+dfrac{1}{bc}+dfrac{1}{ac}=3$$
so use Cauchy-Schwarz inequality we have
$$(ab+bc+ac)left(dfrac{1}{ab}+dfrac{1}{bc}+dfrac{1}{ac}right)ge 9$$
so we have
$$ab+bc+acge 3$$







inequality contest-math symmetry






share|cite|improve this question















share|cite|improve this question













share|cite|improve this question




share|cite|improve this question








edited Jul 6 '18 at 9:46







function sug

















asked Jul 2 '18 at 13:42









function sugfunction sug

3061438




3061438












  • $begingroup$
    Maybe minimizing the function $sin(a-1)+sin(b-1)+sin(c-1)$ with the condition $a+b+c=3abc$?
    $endgroup$
    – user326159
    Jul 2 '18 at 14:15








  • 4




    $begingroup$
    Why did people vote to close this?
    $endgroup$
    – mathworker21
    Jul 5 '18 at 23:50










  • $begingroup$
    @mathworker21 They want to stop us...
    $endgroup$
    – Yuri Negometyanov
    Jul 6 '18 at 9:56






  • 2




    $begingroup$
    @mathworker21 : A good question. All three have chosen "This question is missing context or other details." .
    $endgroup$
    – user90369
    Jul 6 '18 at 9:57






  • 1




    $begingroup$
    @mishu Unfortunately this problem cannot be solved as you hoped with two seperate inequalities. The value of $sin(a-1)+sin(b-1)+sin(c-1)$ is negative at $(a,b,c)=(1/4,3/2,14)$ among other points.
    $endgroup$
    – Isaac Browne
    Jul 6 '18 at 22:26


















  • $begingroup$
    Maybe minimizing the function $sin(a-1)+sin(b-1)+sin(c-1)$ with the condition $a+b+c=3abc$?
    $endgroup$
    – user326159
    Jul 2 '18 at 14:15








  • 4




    $begingroup$
    Why did people vote to close this?
    $endgroup$
    – mathworker21
    Jul 5 '18 at 23:50










  • $begingroup$
    @mathworker21 They want to stop us...
    $endgroup$
    – Yuri Negometyanov
    Jul 6 '18 at 9:56






  • 2




    $begingroup$
    @mathworker21 : A good question. All three have chosen "This question is missing context or other details." .
    $endgroup$
    – user90369
    Jul 6 '18 at 9:57






  • 1




    $begingroup$
    @mishu Unfortunately this problem cannot be solved as you hoped with two seperate inequalities. The value of $sin(a-1)+sin(b-1)+sin(c-1)$ is negative at $(a,b,c)=(1/4,3/2,14)$ among other points.
    $endgroup$
    – Isaac Browne
    Jul 6 '18 at 22:26
















$begingroup$
Maybe minimizing the function $sin(a-1)+sin(b-1)+sin(c-1)$ with the condition $a+b+c=3abc$?
$endgroup$
– user326159
Jul 2 '18 at 14:15






$begingroup$
Maybe minimizing the function $sin(a-1)+sin(b-1)+sin(c-1)$ with the condition $a+b+c=3abc$?
$endgroup$
– user326159
Jul 2 '18 at 14:15






4




4




$begingroup$
Why did people vote to close this?
$endgroup$
– mathworker21
Jul 5 '18 at 23:50




$begingroup$
Why did people vote to close this?
$endgroup$
– mathworker21
Jul 5 '18 at 23:50












$begingroup$
@mathworker21 They want to stop us...
$endgroup$
– Yuri Negometyanov
Jul 6 '18 at 9:56




$begingroup$
@mathworker21 They want to stop us...
$endgroup$
– Yuri Negometyanov
Jul 6 '18 at 9:56




2




2




$begingroup$
@mathworker21 : A good question. All three have chosen "This question is missing context or other details." .
$endgroup$
– user90369
Jul 6 '18 at 9:57




$begingroup$
@mathworker21 : A good question. All three have chosen "This question is missing context or other details." .
$endgroup$
– user90369
Jul 6 '18 at 9:57




1




1




$begingroup$
@mishu Unfortunately this problem cannot be solved as you hoped with two seperate inequalities. The value of $sin(a-1)+sin(b-1)+sin(c-1)$ is negative at $(a,b,c)=(1/4,3/2,14)$ among other points.
$endgroup$
– Isaac Browne
Jul 6 '18 at 22:26




$begingroup$
@mishu Unfortunately this problem cannot be solved as you hoped with two seperate inequalities. The value of $sin(a-1)+sin(b-1)+sin(c-1)$ is negative at $(a,b,c)=(1/4,3/2,14)$ among other points.
$endgroup$
– Isaac Browne
Jul 6 '18 at 22:26










3 Answers
3






active

oldest

votes


















1












$begingroup$

Alright, I think I've finally got a solution, but be warned, it is not a particularly nice one. First off, let's establish the minimum within $0<a,b,cleq2$, (namely $a=b=c=1$) by using lagrange multipliers, then we'll worry about the case where the numbers can be greater than $2$. So, let's call our constraint function
$$g(a,b,c)=a+b+c-3abc$$
$$nabla g(a,b,c) = [1-3bc,1-3ca,1-3ab]$$
And our function to be minimized
$$f(a,b,c)=ab+bc+ca+sin(a-1)+sin(b-1)+sin(c-1)$$
$$nabla f(a,b,c) = [b+c+cos(a-1),c+a+cos(b-1),a+b+cos(c-1)]$$
Now using lagrange multipliers we can write
$$lambda (1-3bc)=b+c+cos(a-1)$$
and so on, but to make things easier and symmetrical, let's make a new variable $k=a+b+c=3abcgeq 3$ so we can write instead
$$lambda =h(a)=frac{k-a+cos(a-1)}{1-frac{k}{a}}=-a+frac{cos(a-1)}{1-frac{k}{a}}$$
Note that we can write $lambda$ in this way replacing $a$ with $b$ or $c$ as it is nice and symmetrical. In order to show that there is one unique solution, I came up with the following argument (which is the only vaguely interesting part of this entire proof, so stop reading after this if you don't want to be bothered with the rest of this mess). If $h(a)$ is a strictly decreasing (or increasing) function over our interval $0<aleq2$, then in order for $lambda$ to be constant, we must have $a=b=c$, and if $a=b=c$, then we have $a+a+a=3aaa$ with the only positive solution being $a=b=c=1$ giving $f(a,b,c)=3$ as our local minimum.



MESSY PART Now to show that $h(x)$ is in fact strictly decreasing over our interval $(0,2]$, so let's take the derivative! We want
$$0>h'(x)=-1+frac{-(1-frac{k}{a})sin(a-1)-cos(a-1)frac{k}{a^2}}{(1-frac{k}{a})^2}$$
This inequality is implied by
$$0>(ka-a^2)sin(a-1) - kcos(a-1) - (k-a)^2$$
which is in turn implied by
$$0>(k-a)asin(1)-kcos(1) - (k-a)^2$$
since $sin(1)$ is the maximum of $sin(a-1)$ and $cos(1)$ is the maximum of $-cos(a-1)$ over the interval $a in (0,2]$
Now this expression is strictly increasing over the interval since
$$0<(k-2a)sin(1)-2a+2k$$
because
$$0<-1-4+6 < (3-4)sin(1)-4+6$$



So our maximum lies at $a=2$, and when we check that against our previous inequality, we get
$$0>(2k-4)sin(1)-kcos(1) - (k-2)^2$$
And this function is strictly decreasing for $k>3$ since
$$0geq 2+4-2 cdot 3 - 0 > 2sin(1)+4-2k - cos(1)$$
and if we plug in $k=3$ we get a true inequality, that is
$$0>(2 cdot 3-4)sin(1)-3cos(1)- (3-2)^2$$
This can be easily proved by bounding $sin(1)$ and $cos(1)$ with a short Taylor expansion, namely $sin(1)=1$ (ending on a positive for the upper bound) and $cos(1)=1-frac{1}{2}$ (ending on a negative for the lower bound). So we get
$$0>2 - 3 frac{1}{2} - 1$$
Thus our original inequality holds and the unique extremum is $a=b=c=1$ for $0<a,b,c<2$



And now we must check the boundary cases. Note that this function and constraint is completely symmetrical so we only need to check where $c=2$. Rewriting the inequality we now must prove
$$ab+2(a+b)+sin(a-1)+sin(b-1)+sin(1)>3$$
with the constraints $a+b+2=6ab$ and $a,b leq 2$



We can rewrite the first constraint as $b = frac{a+2}{6a-1} = 1/6 + frac{2+1/6}{6a-1}$ and we now we can refine the constraint on $a$ and $b$ since we know that this function of $a$ is decreasing for $a>1/6$ so we know the lowest value of $b$ (and thus $a$) is when $a=2$, so we also have $a,b geq frac{4}{11}$
Now the minimum of $ab+2(a+b)=frac{1}{3}+frac{13}{6}(a+frac{1}{6} + frac{13/6}{6a-1})$ and thus really just $a+b$ can be found by taking the derivative yet again. Doing so we get
$$D_a(a+frac{1}{3}+frac{13/6}{6a-1})=1-frac{13}{(6a-1)^2}$$
And setting it equal to $0$ we get the solution $a=b=frac{1+sqrt{13}}{6}$. Now for $a>frac{1+sqrt{13}}{6}$, since $6a-1$ is clearly increasing, $D_a$ will be postive, and since this is symmetric, this means that $a=b=frac{1+sqrt{13}}{6}$ must be the minimum.



Now that we have this, let's plug it in! Doing so we get our minimum to be
$$(2+frac{1}{6})(2frac{1+sqrt{13}}{6})+frac{1}{3}>frac{13}{6} cdot frac{1+3.6}{3}+frac{1}{3}=frac{13 cdot 23 + 30}{90} = frac{329}{90} = 3 + frac{59}{90} $$
And now we take our bounds of $a$ to take on the worst-case-scenario for the sinusoidal part of the expression
$$2sin(frac{4}{11}-1)+sin(1) > -frac{10}{11}+1-frac{1}{6} > -frac{12-11-2}{12}=-frac{1}{12}$$
And now we have proved the inequality around the boundary regions, since $frac{59}{90}>frac{1}{12}$



Now that that's done, in a similar vein we can show this for our first interval $2<c leq 3$. Firstly, we assume WLOG that $c geq a,b$ and now we can establish new bounds for $a$ and $b$ by our constraint $b = frac{a+c}{3ca-1}$ to get $frac{2c}{3c^2-1} leq a,b leq c$. Note that this minimum is strictly decreasing for $c>2$ since the denominator increases much faster than the numerator, so the lowest point we find is at $c=3$, and that is $frac{3}{13}$. Now let's again find the minimum of our expression $a+b$ using the derivative again since
$$D_a left(a+frac{1}{3}+frac{c+frac{1}{3c}}{3ca-1}right)=1-frac{3c^2+1}{(3ca-1)^2}$$
And setting it equal to $0$ again we get the solution $a=b=frac{1+sqrt{1+3c^2}}{3c}$. This is again the minimum since because $3ca-1$ is increasing, $D_a$ will be positive for $a>frac{1+sqrt{1+3c^2}}{3c}$, and the function is still symmetric. The minimum of $ab+c(a+b)=(c+frac{1}{3c})(a+b)$ is thus
$$s(c) = left(c+frac{1}{3c}right)left(2frac{frac{1}{c}+sqrt{frac{1}{c^2}+3}}{3}right)+frac{1}{3}$$
Note that this $s(x)$ is increasing for $c>2$ since
$$D_c (s(c)) = frac{2}{3}left(1-frac{1}{3c^2}right)left(frac{1}{c} + frac{2}{3}sqrt{frac{1}{c^2}+3}right) - left(c+frac{1}{3c}right)left(frac{1}{c^2}+frac{1}{c^3} frac{1}{sqrt{frac{1}{c^2}+3}}right) > 0 $$
$$iff left(c^2-frac{1}{3}right)left(1+sqrt{1+3c^2}right)-left(c^2+frac{1}{3}right)left(1+frac{1}{sqrt{1+3c^2}}right)>0$$
$$iff -frac{2}{3}+c^2left(sqrt{1+3c^2}-frac{1}{sqrt{1+3c^2}}right)-frac{1}{3}left(sqrt{1+3c^2}+frac{1}{sqrt{1+3c^2}}right) > 0$$
And since $c>2$, this is implied by
$$-frac{2}{3}+c^2left(sqrt{1+3c^2}-frac{1}{3}right)-frac{1}{3}left(sqrt{1+3c^2}+frac{1}{3}right) > 0$$
$$iff -frac{8}{9}+left(c^2-frac{1}{3}right)left(sqrt{1+3c^2}-frac{1}{3}right) > 0$$
And now, since we finally have two increasing functions multiplied together, we can replace each with their lowest value to prove the inequality. Thus since
$$left(4-frac{1}{3}right)left(sqrt{13}-frac{1}{3}right)-frac{8}{9}>0$$
$s(c)$ is increasing.



Now we have shown that $s(c)$ is increasing and that the minimum of $a,b$ and thus the minimum of $sin(a-1)$ since $0<a<3$. To prove this inequality for $2<c<3$, all that is left is to do is plug in our worst case scenario. So we have to show
$$s(2)-2sin(frac{3}{13}-1)+sin(1)>3$$
Since $sin(1)$ is the minimum of $sin(c-1)$ over $2<c leq 3$. We already calculated $s(2)$ so now we use a few taylor expansions to obtain that this inequality is implied by
$$3+frac{59}{90}-2left(frac{10}{13}-frac{left(frac{10}{13}right)^3}{3!}+frac{left(frac{10}{13}right)^5}{5!}right)+1-frac{1}{6} > 3$$
Which you can do out yourself if you'd like. At this point I will stop doing out these calculations as they make the answer tedious and long.



For our next interval $3<c<1 + pi$ we can use a better bound for the minimum and maximum of $a,b$ since if $ab+c(a+b)>5$ then it follows that $$ab+c(a+b) + sin(a-1)+ sin(b-1) + sin(c-1) > 5 - 1 - 1 + 0 = 3$$. So all cases where $ab+c(a+b)>5$ are accounted for and this imposing at least the maximum
$a,b<a+b<frac{5}{c}$, and this implies the minimum $a,b>frac{2(5/c)}{3c(5/c)-1}=frac{5}{7c}$. Note that this function is still a decreasing one and now we have over the interval $3<c<1+pi<frac{7+22}{7}$ that $a,b > frac{5}{29}$. Using this, we can now prove our inequality by showing
$$3 < ab+c(a+b) + sin(a-1) + sin(b-1) + sin(c-1) < s(3) + 2sin(5/29-1) < s(3) -48/29 approx 3.02784 $$
And this can be trivially shown by bounding $s(3)$ as well.



Next interval: $1+pi < c < 1 + pi + 1$. We can no longer use the bound from the previous problem, but we can at this point just say
$$s(1+pi) + sin(a-1)+sin(b-1)+sin(c-1) > s(4) -3sin(1) approx 3.25 > 3$$
This can again easily be done using a quick taylor expansion, and that $s(4)=frac{52}{9}$



Finally for our interval $c>2+pi$ we finally have $s(2+pi)>6$, so we are done.



Note: There is probably at least one mistake in here, given the length, so feel free to leave comments.






share|cite|improve this answer









$endgroup$





















    1












    $begingroup$



    $$mathbf{color{brown}{Analysis of the issue condition.}}$$



    Let WLOG
    $$0<ale ble c,tag1$$
    $$a+b+c=3abc,tag2$$
    then
    $$c+a=(3ac-1)b,quad (3ac-1)ale a+cle(3ac-1)c,$$
    $$begin{cases}
    (3a^2-1)cle2a\[4pt]
    (3c^2-1)ale2c\[4pt]
    0<ale ble c,
    end{cases}rightarrow
    begin{cases}
    dfrac1{sqrt3} < ale dfrac{2c}{3c^2-1}\
    ale cledfrac{2a}{3a^2-1}\
    dfrac{2c}{3c^2-1}le ale c\
    end{cases}rightarrow
    begin{cases}
    dfrac1{sqrt3}< a\[4pt]
    (sqrt3c-1)^2le2\[4pt]
    3a^2-1le 2le 3c^2-1\[4pt]
    end{cases}$$
    $$boxed {dfrac1{sqrt3}<ale1le cledfrac{sqrt2+1}{sqrt3},quad b=dfrac{a+c}{3ac-1}.}tag3$$
    Besides, as shown in OP,
    $$dfrac13left(dfrac1{ab}+dfrac1{bc}+dfrac1{ca}right)=1.$$
    Using AM-HM, easy to obtain
    $$ab+bc+acge3.tag4$$





    $$mathbf{color{brown}{The trig inequality}}$$



    Let us consider the inequality
    $$sin(a-1) + sin(b-1) + sin(c-1) +lambda(a+b+c -3abc)tag5$$
    under the constraints $(3).$



    Using Lagrange multipiers method with the function
    $$f(a,b,c,lambda) = sin(a-1) + sin(b-1) + sin(c-1) +lambda(a+b+c -3abc),$$
    the stationary points can be found from the system $f'_a=f'_b=f'_c=f'_lambda=0,$ or
    begin{cases}
    cos(a-1)+ lambda(1-3bc)=0\
    cos(b-1)+ lambda(1-3ca)=0\
    cos(c-1)+ lambda(1-3ab)=0\
    a+b+c=3abc\
    dfrac{sqrt3}3<ale1le cledfrac{sqrt2+1}{sqrt3},\
    end{cases}
    begin{cases}
    dfrac1{3bc-1}cos(a-1) - lambda = 0\
    dfrac1{3ca-1}cos(b-1) - lambda = 0\
    dfrac1{3ab-1}cos(c-1) - lambda = 0\
    a+b+c=3abc\
    dfrac{sqrt3}3<ale1le cledfrac{sqrt2+1}{sqrt3},\
    end{cases}
    begin{cases}
    b =dfrac{a+c}{3ac-1}\[4pt]
    3bc-1=dfrac{3ac+3c^2}{3ac-1}-1 = dfrac{3c^2+1}{3ac-1}\[4pt]
    3ab-1=dfrac{3a^2+3ac}{3ac-1}-1 = dfrac{3a^2+1}{3ac-1}\[4pt]
    dfrac{3ac-1}{3c^2+1}cos(a-1) = dfrac{3ac-1}{3a^2+1}cos(c-1)\[4pt]
    dfrac{3ac-1}{3c^2+1}cos(a-1) = dfrac1{3ac-1}cosleft(dfrac{a+c}{3ac-1}-1right)\[4pt]
    dfrac{sqrt3}3<ale1le cledfrac{sqrt2+1}{sqrt3}.tag6
    end{cases}
    Function $g(x)=(3x^2+1)cos (x-1)$ has positive derivation
    $$g'(x)=6xcos(x-1)-(3x^2+1)sin(x-1)$$
    in $left[dfrac1{sqrt3},dfrac{sqrt2+1}{sqrt3}right],$ so $g(x)$ increases in this interval.



    The equation $(6.4)$ can be rewrited as
    $$g(a)=g(c).$$
    Taking in account constraints $(6.6),$ the system $(6)$ can be satisfied in the single point $$a=1,quad c=g(1)=1.$$
    This gives the single stationary point
    $$a=b=c=1,quad f(a,b,c,lambda)=0.$$



    Note that constraints $(6.6)$ determines the blurring of boundaries between variables and can not be checked.
    On the other hand, checking of the trig inequality in the random point $$left(a,b,c, fright)=left(dfrac23,dfrac65,dfrac43, sindfrac15right)$$ shows that trig inequality holds. Therefore, it is proved.



    Taking in account $(4)$, the issue inequality is proved.






    share|cite|improve this answer











    $endgroup$





















      0












      $begingroup$

      Hint : Prove with Jensen's inequality (apply to the function $f(x)=0.5x^2+sin(x-1)$ ) this with the obvious restriction $ab+bc+caleq6$:




      $$ab+bc+ca+sin(a-1)+sin(b-1)+sin(c-1)$$$$geq$$$$ ab+bc+ca-0.5(a^2+b^2+c^2)+3sin(frac{a+b+c}{3}-1)+1.5(frac{a+b+c}{3})^2$$




      And now a last hint prove this (with the condition $3abc=a+b+c$ and $ab+bc+caleq6$ ):




      $$ab+bc+ca-0.5(a^2+b^2+c^2)+1.5(frac{a+b+c}{3})^2geq 3$$
      And
      $$3sin(frac{a+b+c}{3}-1)geq 0$$







      share|cite|improve this answer









      $endgroup$













        Your Answer





        StackExchange.ifUsing("editor", function () {
        return StackExchange.using("mathjaxEditing", function () {
        StackExchange.MarkdownEditor.creationCallbacks.add(function (editor, postfix) {
        StackExchange.mathjaxEditing.prepareWmdForMathJax(editor, postfix, [["$", "$"], ["\\(","\\)"]]);
        });
        });
        }, "mathjax-editing");

        StackExchange.ready(function() {
        var channelOptions = {
        tags: "".split(" "),
        id: "69"
        };
        initTagRenderer("".split(" "), "".split(" "), channelOptions);

        StackExchange.using("externalEditor", function() {
        // Have to fire editor after snippets, if snippets enabled
        if (StackExchange.settings.snippets.snippetsEnabled) {
        StackExchange.using("snippets", function() {
        createEditor();
        });
        }
        else {
        createEditor();
        }
        });

        function createEditor() {
        StackExchange.prepareEditor({
        heartbeatType: 'answer',
        autoActivateHeartbeat: false,
        convertImagesToLinks: true,
        noModals: true,
        showLowRepImageUploadWarning: true,
        reputationToPostImages: 10,
        bindNavPrevention: true,
        postfix: "",
        imageUploader: {
        brandingHtml: "Powered by u003ca class="icon-imgur-white" href="https://imgur.com/"u003eu003c/au003e",
        contentPolicyHtml: "User contributions licensed under u003ca href="https://creativecommons.org/licenses/by-sa/3.0/"u003ecc by-sa 3.0 with attribution requiredu003c/au003e u003ca href="https://stackoverflow.com/legal/content-policy"u003e(content policy)u003c/au003e",
        allowUrls: true
        },
        noCode: true, onDemand: true,
        discardSelector: ".discard-answer"
        ,immediatelyShowMarkdownHelp:true
        });


        }
        });














        draft saved

        draft discarded


















        StackExchange.ready(
        function () {
        StackExchange.openid.initPostLogin('.new-post-login', 'https%3a%2f%2fmath.stackexchange.com%2fquestions%2f2838516%2fshow-this-inequality-abbcac-sina-1-sinb-1-sinc-1-ge-3%23new-answer', 'question_page');
        }
        );

        Post as a guest















        Required, but never shown

























        3 Answers
        3






        active

        oldest

        votes








        3 Answers
        3






        active

        oldest

        votes









        active

        oldest

        votes






        active

        oldest

        votes









        1












        $begingroup$

        Alright, I think I've finally got a solution, but be warned, it is not a particularly nice one. First off, let's establish the minimum within $0<a,b,cleq2$, (namely $a=b=c=1$) by using lagrange multipliers, then we'll worry about the case where the numbers can be greater than $2$. So, let's call our constraint function
        $$g(a,b,c)=a+b+c-3abc$$
        $$nabla g(a,b,c) = [1-3bc,1-3ca,1-3ab]$$
        And our function to be minimized
        $$f(a,b,c)=ab+bc+ca+sin(a-1)+sin(b-1)+sin(c-1)$$
        $$nabla f(a,b,c) = [b+c+cos(a-1),c+a+cos(b-1),a+b+cos(c-1)]$$
        Now using lagrange multipliers we can write
        $$lambda (1-3bc)=b+c+cos(a-1)$$
        and so on, but to make things easier and symmetrical, let's make a new variable $k=a+b+c=3abcgeq 3$ so we can write instead
        $$lambda =h(a)=frac{k-a+cos(a-1)}{1-frac{k}{a}}=-a+frac{cos(a-1)}{1-frac{k}{a}}$$
        Note that we can write $lambda$ in this way replacing $a$ with $b$ or $c$ as it is nice and symmetrical. In order to show that there is one unique solution, I came up with the following argument (which is the only vaguely interesting part of this entire proof, so stop reading after this if you don't want to be bothered with the rest of this mess). If $h(a)$ is a strictly decreasing (or increasing) function over our interval $0<aleq2$, then in order for $lambda$ to be constant, we must have $a=b=c$, and if $a=b=c$, then we have $a+a+a=3aaa$ with the only positive solution being $a=b=c=1$ giving $f(a,b,c)=3$ as our local minimum.



        MESSY PART Now to show that $h(x)$ is in fact strictly decreasing over our interval $(0,2]$, so let's take the derivative! We want
        $$0>h'(x)=-1+frac{-(1-frac{k}{a})sin(a-1)-cos(a-1)frac{k}{a^2}}{(1-frac{k}{a})^2}$$
        This inequality is implied by
        $$0>(ka-a^2)sin(a-1) - kcos(a-1) - (k-a)^2$$
        which is in turn implied by
        $$0>(k-a)asin(1)-kcos(1) - (k-a)^2$$
        since $sin(1)$ is the maximum of $sin(a-1)$ and $cos(1)$ is the maximum of $-cos(a-1)$ over the interval $a in (0,2]$
        Now this expression is strictly increasing over the interval since
        $$0<(k-2a)sin(1)-2a+2k$$
        because
        $$0<-1-4+6 < (3-4)sin(1)-4+6$$



        So our maximum lies at $a=2$, and when we check that against our previous inequality, we get
        $$0>(2k-4)sin(1)-kcos(1) - (k-2)^2$$
        And this function is strictly decreasing for $k>3$ since
        $$0geq 2+4-2 cdot 3 - 0 > 2sin(1)+4-2k - cos(1)$$
        and if we plug in $k=3$ we get a true inequality, that is
        $$0>(2 cdot 3-4)sin(1)-3cos(1)- (3-2)^2$$
        This can be easily proved by bounding $sin(1)$ and $cos(1)$ with a short Taylor expansion, namely $sin(1)=1$ (ending on a positive for the upper bound) and $cos(1)=1-frac{1}{2}$ (ending on a negative for the lower bound). So we get
        $$0>2 - 3 frac{1}{2} - 1$$
        Thus our original inequality holds and the unique extremum is $a=b=c=1$ for $0<a,b,c<2$



        And now we must check the boundary cases. Note that this function and constraint is completely symmetrical so we only need to check where $c=2$. Rewriting the inequality we now must prove
        $$ab+2(a+b)+sin(a-1)+sin(b-1)+sin(1)>3$$
        with the constraints $a+b+2=6ab$ and $a,b leq 2$



        We can rewrite the first constraint as $b = frac{a+2}{6a-1} = 1/6 + frac{2+1/6}{6a-1}$ and we now we can refine the constraint on $a$ and $b$ since we know that this function of $a$ is decreasing for $a>1/6$ so we know the lowest value of $b$ (and thus $a$) is when $a=2$, so we also have $a,b geq frac{4}{11}$
        Now the minimum of $ab+2(a+b)=frac{1}{3}+frac{13}{6}(a+frac{1}{6} + frac{13/6}{6a-1})$ and thus really just $a+b$ can be found by taking the derivative yet again. Doing so we get
        $$D_a(a+frac{1}{3}+frac{13/6}{6a-1})=1-frac{13}{(6a-1)^2}$$
        And setting it equal to $0$ we get the solution $a=b=frac{1+sqrt{13}}{6}$. Now for $a>frac{1+sqrt{13}}{6}$, since $6a-1$ is clearly increasing, $D_a$ will be postive, and since this is symmetric, this means that $a=b=frac{1+sqrt{13}}{6}$ must be the minimum.



        Now that we have this, let's plug it in! Doing so we get our minimum to be
        $$(2+frac{1}{6})(2frac{1+sqrt{13}}{6})+frac{1}{3}>frac{13}{6} cdot frac{1+3.6}{3}+frac{1}{3}=frac{13 cdot 23 + 30}{90} = frac{329}{90} = 3 + frac{59}{90} $$
        And now we take our bounds of $a$ to take on the worst-case-scenario for the sinusoidal part of the expression
        $$2sin(frac{4}{11}-1)+sin(1) > -frac{10}{11}+1-frac{1}{6} > -frac{12-11-2}{12}=-frac{1}{12}$$
        And now we have proved the inequality around the boundary regions, since $frac{59}{90}>frac{1}{12}$



        Now that that's done, in a similar vein we can show this for our first interval $2<c leq 3$. Firstly, we assume WLOG that $c geq a,b$ and now we can establish new bounds for $a$ and $b$ by our constraint $b = frac{a+c}{3ca-1}$ to get $frac{2c}{3c^2-1} leq a,b leq c$. Note that this minimum is strictly decreasing for $c>2$ since the denominator increases much faster than the numerator, so the lowest point we find is at $c=3$, and that is $frac{3}{13}$. Now let's again find the minimum of our expression $a+b$ using the derivative again since
        $$D_a left(a+frac{1}{3}+frac{c+frac{1}{3c}}{3ca-1}right)=1-frac{3c^2+1}{(3ca-1)^2}$$
        And setting it equal to $0$ again we get the solution $a=b=frac{1+sqrt{1+3c^2}}{3c}$. This is again the minimum since because $3ca-1$ is increasing, $D_a$ will be positive for $a>frac{1+sqrt{1+3c^2}}{3c}$, and the function is still symmetric. The minimum of $ab+c(a+b)=(c+frac{1}{3c})(a+b)$ is thus
        $$s(c) = left(c+frac{1}{3c}right)left(2frac{frac{1}{c}+sqrt{frac{1}{c^2}+3}}{3}right)+frac{1}{3}$$
        Note that this $s(x)$ is increasing for $c>2$ since
        $$D_c (s(c)) = frac{2}{3}left(1-frac{1}{3c^2}right)left(frac{1}{c} + frac{2}{3}sqrt{frac{1}{c^2}+3}right) - left(c+frac{1}{3c}right)left(frac{1}{c^2}+frac{1}{c^3} frac{1}{sqrt{frac{1}{c^2}+3}}right) > 0 $$
        $$iff left(c^2-frac{1}{3}right)left(1+sqrt{1+3c^2}right)-left(c^2+frac{1}{3}right)left(1+frac{1}{sqrt{1+3c^2}}right)>0$$
        $$iff -frac{2}{3}+c^2left(sqrt{1+3c^2}-frac{1}{sqrt{1+3c^2}}right)-frac{1}{3}left(sqrt{1+3c^2}+frac{1}{sqrt{1+3c^2}}right) > 0$$
        And since $c>2$, this is implied by
        $$-frac{2}{3}+c^2left(sqrt{1+3c^2}-frac{1}{3}right)-frac{1}{3}left(sqrt{1+3c^2}+frac{1}{3}right) > 0$$
        $$iff -frac{8}{9}+left(c^2-frac{1}{3}right)left(sqrt{1+3c^2}-frac{1}{3}right) > 0$$
        And now, since we finally have two increasing functions multiplied together, we can replace each with their lowest value to prove the inequality. Thus since
        $$left(4-frac{1}{3}right)left(sqrt{13}-frac{1}{3}right)-frac{8}{9}>0$$
        $s(c)$ is increasing.



        Now we have shown that $s(c)$ is increasing and that the minimum of $a,b$ and thus the minimum of $sin(a-1)$ since $0<a<3$. To prove this inequality for $2<c<3$, all that is left is to do is plug in our worst case scenario. So we have to show
        $$s(2)-2sin(frac{3}{13}-1)+sin(1)>3$$
        Since $sin(1)$ is the minimum of $sin(c-1)$ over $2<c leq 3$. We already calculated $s(2)$ so now we use a few taylor expansions to obtain that this inequality is implied by
        $$3+frac{59}{90}-2left(frac{10}{13}-frac{left(frac{10}{13}right)^3}{3!}+frac{left(frac{10}{13}right)^5}{5!}right)+1-frac{1}{6} > 3$$
        Which you can do out yourself if you'd like. At this point I will stop doing out these calculations as they make the answer tedious and long.



        For our next interval $3<c<1 + pi$ we can use a better bound for the minimum and maximum of $a,b$ since if $ab+c(a+b)>5$ then it follows that $$ab+c(a+b) + sin(a-1)+ sin(b-1) + sin(c-1) > 5 - 1 - 1 + 0 = 3$$. So all cases where $ab+c(a+b)>5$ are accounted for and this imposing at least the maximum
        $a,b<a+b<frac{5}{c}$, and this implies the minimum $a,b>frac{2(5/c)}{3c(5/c)-1}=frac{5}{7c}$. Note that this function is still a decreasing one and now we have over the interval $3<c<1+pi<frac{7+22}{7}$ that $a,b > frac{5}{29}$. Using this, we can now prove our inequality by showing
        $$3 < ab+c(a+b) + sin(a-1) + sin(b-1) + sin(c-1) < s(3) + 2sin(5/29-1) < s(3) -48/29 approx 3.02784 $$
        And this can be trivially shown by bounding $s(3)$ as well.



        Next interval: $1+pi < c < 1 + pi + 1$. We can no longer use the bound from the previous problem, but we can at this point just say
        $$s(1+pi) + sin(a-1)+sin(b-1)+sin(c-1) > s(4) -3sin(1) approx 3.25 > 3$$
        This can again easily be done using a quick taylor expansion, and that $s(4)=frac{52}{9}$



        Finally for our interval $c>2+pi$ we finally have $s(2+pi)>6$, so we are done.



        Note: There is probably at least one mistake in here, given the length, so feel free to leave comments.






        share|cite|improve this answer









        $endgroup$


















          1












          $begingroup$

          Alright, I think I've finally got a solution, but be warned, it is not a particularly nice one. First off, let's establish the minimum within $0<a,b,cleq2$, (namely $a=b=c=1$) by using lagrange multipliers, then we'll worry about the case where the numbers can be greater than $2$. So, let's call our constraint function
          $$g(a,b,c)=a+b+c-3abc$$
          $$nabla g(a,b,c) = [1-3bc,1-3ca,1-3ab]$$
          And our function to be minimized
          $$f(a,b,c)=ab+bc+ca+sin(a-1)+sin(b-1)+sin(c-1)$$
          $$nabla f(a,b,c) = [b+c+cos(a-1),c+a+cos(b-1),a+b+cos(c-1)]$$
          Now using lagrange multipliers we can write
          $$lambda (1-3bc)=b+c+cos(a-1)$$
          and so on, but to make things easier and symmetrical, let's make a new variable $k=a+b+c=3abcgeq 3$ so we can write instead
          $$lambda =h(a)=frac{k-a+cos(a-1)}{1-frac{k}{a}}=-a+frac{cos(a-1)}{1-frac{k}{a}}$$
          Note that we can write $lambda$ in this way replacing $a$ with $b$ or $c$ as it is nice and symmetrical. In order to show that there is one unique solution, I came up with the following argument (which is the only vaguely interesting part of this entire proof, so stop reading after this if you don't want to be bothered with the rest of this mess). If $h(a)$ is a strictly decreasing (or increasing) function over our interval $0<aleq2$, then in order for $lambda$ to be constant, we must have $a=b=c$, and if $a=b=c$, then we have $a+a+a=3aaa$ with the only positive solution being $a=b=c=1$ giving $f(a,b,c)=3$ as our local minimum.



          MESSY PART Now to show that $h(x)$ is in fact strictly decreasing over our interval $(0,2]$, so let's take the derivative! We want
          $$0>h'(x)=-1+frac{-(1-frac{k}{a})sin(a-1)-cos(a-1)frac{k}{a^2}}{(1-frac{k}{a})^2}$$
          This inequality is implied by
          $$0>(ka-a^2)sin(a-1) - kcos(a-1) - (k-a)^2$$
          which is in turn implied by
          $$0>(k-a)asin(1)-kcos(1) - (k-a)^2$$
          since $sin(1)$ is the maximum of $sin(a-1)$ and $cos(1)$ is the maximum of $-cos(a-1)$ over the interval $a in (0,2]$
          Now this expression is strictly increasing over the interval since
          $$0<(k-2a)sin(1)-2a+2k$$
          because
          $$0<-1-4+6 < (3-4)sin(1)-4+6$$



          So our maximum lies at $a=2$, and when we check that against our previous inequality, we get
          $$0>(2k-4)sin(1)-kcos(1) - (k-2)^2$$
          And this function is strictly decreasing for $k>3$ since
          $$0geq 2+4-2 cdot 3 - 0 > 2sin(1)+4-2k - cos(1)$$
          and if we plug in $k=3$ we get a true inequality, that is
          $$0>(2 cdot 3-4)sin(1)-3cos(1)- (3-2)^2$$
          This can be easily proved by bounding $sin(1)$ and $cos(1)$ with a short Taylor expansion, namely $sin(1)=1$ (ending on a positive for the upper bound) and $cos(1)=1-frac{1}{2}$ (ending on a negative for the lower bound). So we get
          $$0>2 - 3 frac{1}{2} - 1$$
          Thus our original inequality holds and the unique extremum is $a=b=c=1$ for $0<a,b,c<2$



          And now we must check the boundary cases. Note that this function and constraint is completely symmetrical so we only need to check where $c=2$. Rewriting the inequality we now must prove
          $$ab+2(a+b)+sin(a-1)+sin(b-1)+sin(1)>3$$
          with the constraints $a+b+2=6ab$ and $a,b leq 2$



          We can rewrite the first constraint as $b = frac{a+2}{6a-1} = 1/6 + frac{2+1/6}{6a-1}$ and we now we can refine the constraint on $a$ and $b$ since we know that this function of $a$ is decreasing for $a>1/6$ so we know the lowest value of $b$ (and thus $a$) is when $a=2$, so we also have $a,b geq frac{4}{11}$
          Now the minimum of $ab+2(a+b)=frac{1}{3}+frac{13}{6}(a+frac{1}{6} + frac{13/6}{6a-1})$ and thus really just $a+b$ can be found by taking the derivative yet again. Doing so we get
          $$D_a(a+frac{1}{3}+frac{13/6}{6a-1})=1-frac{13}{(6a-1)^2}$$
          And setting it equal to $0$ we get the solution $a=b=frac{1+sqrt{13}}{6}$. Now for $a>frac{1+sqrt{13}}{6}$, since $6a-1$ is clearly increasing, $D_a$ will be postive, and since this is symmetric, this means that $a=b=frac{1+sqrt{13}}{6}$ must be the minimum.



          Now that we have this, let's plug it in! Doing so we get our minimum to be
          $$(2+frac{1}{6})(2frac{1+sqrt{13}}{6})+frac{1}{3}>frac{13}{6} cdot frac{1+3.6}{3}+frac{1}{3}=frac{13 cdot 23 + 30}{90} = frac{329}{90} = 3 + frac{59}{90} $$
          And now we take our bounds of $a$ to take on the worst-case-scenario for the sinusoidal part of the expression
          $$2sin(frac{4}{11}-1)+sin(1) > -frac{10}{11}+1-frac{1}{6} > -frac{12-11-2}{12}=-frac{1}{12}$$
          And now we have proved the inequality around the boundary regions, since $frac{59}{90}>frac{1}{12}$



          Now that that's done, in a similar vein we can show this for our first interval $2<c leq 3$. Firstly, we assume WLOG that $c geq a,b$ and now we can establish new bounds for $a$ and $b$ by our constraint $b = frac{a+c}{3ca-1}$ to get $frac{2c}{3c^2-1} leq a,b leq c$. Note that this minimum is strictly decreasing for $c>2$ since the denominator increases much faster than the numerator, so the lowest point we find is at $c=3$, and that is $frac{3}{13}$. Now let's again find the minimum of our expression $a+b$ using the derivative again since
          $$D_a left(a+frac{1}{3}+frac{c+frac{1}{3c}}{3ca-1}right)=1-frac{3c^2+1}{(3ca-1)^2}$$
          And setting it equal to $0$ again we get the solution $a=b=frac{1+sqrt{1+3c^2}}{3c}$. This is again the minimum since because $3ca-1$ is increasing, $D_a$ will be positive for $a>frac{1+sqrt{1+3c^2}}{3c}$, and the function is still symmetric. The minimum of $ab+c(a+b)=(c+frac{1}{3c})(a+b)$ is thus
          $$s(c) = left(c+frac{1}{3c}right)left(2frac{frac{1}{c}+sqrt{frac{1}{c^2}+3}}{3}right)+frac{1}{3}$$
          Note that this $s(x)$ is increasing for $c>2$ since
          $$D_c (s(c)) = frac{2}{3}left(1-frac{1}{3c^2}right)left(frac{1}{c} + frac{2}{3}sqrt{frac{1}{c^2}+3}right) - left(c+frac{1}{3c}right)left(frac{1}{c^2}+frac{1}{c^3} frac{1}{sqrt{frac{1}{c^2}+3}}right) > 0 $$
          $$iff left(c^2-frac{1}{3}right)left(1+sqrt{1+3c^2}right)-left(c^2+frac{1}{3}right)left(1+frac{1}{sqrt{1+3c^2}}right)>0$$
          $$iff -frac{2}{3}+c^2left(sqrt{1+3c^2}-frac{1}{sqrt{1+3c^2}}right)-frac{1}{3}left(sqrt{1+3c^2}+frac{1}{sqrt{1+3c^2}}right) > 0$$
          And since $c>2$, this is implied by
          $$-frac{2}{3}+c^2left(sqrt{1+3c^2}-frac{1}{3}right)-frac{1}{3}left(sqrt{1+3c^2}+frac{1}{3}right) > 0$$
          $$iff -frac{8}{9}+left(c^2-frac{1}{3}right)left(sqrt{1+3c^2}-frac{1}{3}right) > 0$$
          And now, since we finally have two increasing functions multiplied together, we can replace each with their lowest value to prove the inequality. Thus since
          $$left(4-frac{1}{3}right)left(sqrt{13}-frac{1}{3}right)-frac{8}{9}>0$$
          $s(c)$ is increasing.



          Now we have shown that $s(c)$ is increasing and that the minimum of $a,b$ and thus the minimum of $sin(a-1)$ since $0<a<3$. To prove this inequality for $2<c<3$, all that is left is to do is plug in our worst case scenario. So we have to show
          $$s(2)-2sin(frac{3}{13}-1)+sin(1)>3$$
          Since $sin(1)$ is the minimum of $sin(c-1)$ over $2<c leq 3$. We already calculated $s(2)$ so now we use a few taylor expansions to obtain that this inequality is implied by
          $$3+frac{59}{90}-2left(frac{10}{13}-frac{left(frac{10}{13}right)^3}{3!}+frac{left(frac{10}{13}right)^5}{5!}right)+1-frac{1}{6} > 3$$
          Which you can do out yourself if you'd like. At this point I will stop doing out these calculations as they make the answer tedious and long.



          For our next interval $3<c<1 + pi$ we can use a better bound for the minimum and maximum of $a,b$ since if $ab+c(a+b)>5$ then it follows that $$ab+c(a+b) + sin(a-1)+ sin(b-1) + sin(c-1) > 5 - 1 - 1 + 0 = 3$$. So all cases where $ab+c(a+b)>5$ are accounted for and this imposing at least the maximum
          $a,b<a+b<frac{5}{c}$, and this implies the minimum $a,b>frac{2(5/c)}{3c(5/c)-1}=frac{5}{7c}$. Note that this function is still a decreasing one and now we have over the interval $3<c<1+pi<frac{7+22}{7}$ that $a,b > frac{5}{29}$. Using this, we can now prove our inequality by showing
          $$3 < ab+c(a+b) + sin(a-1) + sin(b-1) + sin(c-1) < s(3) + 2sin(5/29-1) < s(3) -48/29 approx 3.02784 $$
          And this can be trivially shown by bounding $s(3)$ as well.



          Next interval: $1+pi < c < 1 + pi + 1$. We can no longer use the bound from the previous problem, but we can at this point just say
          $$s(1+pi) + sin(a-1)+sin(b-1)+sin(c-1) > s(4) -3sin(1) approx 3.25 > 3$$
          This can again easily be done using a quick taylor expansion, and that $s(4)=frac{52}{9}$



          Finally for our interval $c>2+pi$ we finally have $s(2+pi)>6$, so we are done.



          Note: There is probably at least one mistake in here, given the length, so feel free to leave comments.






          share|cite|improve this answer









          $endgroup$
















            1












            1








            1





            $begingroup$

            Alright, I think I've finally got a solution, but be warned, it is not a particularly nice one. First off, let's establish the minimum within $0<a,b,cleq2$, (namely $a=b=c=1$) by using lagrange multipliers, then we'll worry about the case where the numbers can be greater than $2$. So, let's call our constraint function
            $$g(a,b,c)=a+b+c-3abc$$
            $$nabla g(a,b,c) = [1-3bc,1-3ca,1-3ab]$$
            And our function to be minimized
            $$f(a,b,c)=ab+bc+ca+sin(a-1)+sin(b-1)+sin(c-1)$$
            $$nabla f(a,b,c) = [b+c+cos(a-1),c+a+cos(b-1),a+b+cos(c-1)]$$
            Now using lagrange multipliers we can write
            $$lambda (1-3bc)=b+c+cos(a-1)$$
            and so on, but to make things easier and symmetrical, let's make a new variable $k=a+b+c=3abcgeq 3$ so we can write instead
            $$lambda =h(a)=frac{k-a+cos(a-1)}{1-frac{k}{a}}=-a+frac{cos(a-1)}{1-frac{k}{a}}$$
            Note that we can write $lambda$ in this way replacing $a$ with $b$ or $c$ as it is nice and symmetrical. In order to show that there is one unique solution, I came up with the following argument (which is the only vaguely interesting part of this entire proof, so stop reading after this if you don't want to be bothered with the rest of this mess). If $h(a)$ is a strictly decreasing (or increasing) function over our interval $0<aleq2$, then in order for $lambda$ to be constant, we must have $a=b=c$, and if $a=b=c$, then we have $a+a+a=3aaa$ with the only positive solution being $a=b=c=1$ giving $f(a,b,c)=3$ as our local minimum.



            MESSY PART Now to show that $h(x)$ is in fact strictly decreasing over our interval $(0,2]$, so let's take the derivative! We want
            $$0>h'(x)=-1+frac{-(1-frac{k}{a})sin(a-1)-cos(a-1)frac{k}{a^2}}{(1-frac{k}{a})^2}$$
            This inequality is implied by
            $$0>(ka-a^2)sin(a-1) - kcos(a-1) - (k-a)^2$$
            which is in turn implied by
            $$0>(k-a)asin(1)-kcos(1) - (k-a)^2$$
            since $sin(1)$ is the maximum of $sin(a-1)$ and $cos(1)$ is the maximum of $-cos(a-1)$ over the interval $a in (0,2]$
            Now this expression is strictly increasing over the interval since
            $$0<(k-2a)sin(1)-2a+2k$$
            because
            $$0<-1-4+6 < (3-4)sin(1)-4+6$$



            So our maximum lies at $a=2$, and when we check that against our previous inequality, we get
            $$0>(2k-4)sin(1)-kcos(1) - (k-2)^2$$
            And this function is strictly decreasing for $k>3$ since
            $$0geq 2+4-2 cdot 3 - 0 > 2sin(1)+4-2k - cos(1)$$
            and if we plug in $k=3$ we get a true inequality, that is
            $$0>(2 cdot 3-4)sin(1)-3cos(1)- (3-2)^2$$
            This can be easily proved by bounding $sin(1)$ and $cos(1)$ with a short Taylor expansion, namely $sin(1)=1$ (ending on a positive for the upper bound) and $cos(1)=1-frac{1}{2}$ (ending on a negative for the lower bound). So we get
            $$0>2 - 3 frac{1}{2} - 1$$
            Thus our original inequality holds and the unique extremum is $a=b=c=1$ for $0<a,b,c<2$



            And now we must check the boundary cases. Note that this function and constraint is completely symmetrical so we only need to check where $c=2$. Rewriting the inequality we now must prove
            $$ab+2(a+b)+sin(a-1)+sin(b-1)+sin(1)>3$$
            with the constraints $a+b+2=6ab$ and $a,b leq 2$



            We can rewrite the first constraint as $b = frac{a+2}{6a-1} = 1/6 + frac{2+1/6}{6a-1}$ and we now we can refine the constraint on $a$ and $b$ since we know that this function of $a$ is decreasing for $a>1/6$ so we know the lowest value of $b$ (and thus $a$) is when $a=2$, so we also have $a,b geq frac{4}{11}$
            Now the minimum of $ab+2(a+b)=frac{1}{3}+frac{13}{6}(a+frac{1}{6} + frac{13/6}{6a-1})$ and thus really just $a+b$ can be found by taking the derivative yet again. Doing so we get
            $$D_a(a+frac{1}{3}+frac{13/6}{6a-1})=1-frac{13}{(6a-1)^2}$$
            And setting it equal to $0$ we get the solution $a=b=frac{1+sqrt{13}}{6}$. Now for $a>frac{1+sqrt{13}}{6}$, since $6a-1$ is clearly increasing, $D_a$ will be postive, and since this is symmetric, this means that $a=b=frac{1+sqrt{13}}{6}$ must be the minimum.



            Now that we have this, let's plug it in! Doing so we get our minimum to be
            $$(2+frac{1}{6})(2frac{1+sqrt{13}}{6})+frac{1}{3}>frac{13}{6} cdot frac{1+3.6}{3}+frac{1}{3}=frac{13 cdot 23 + 30}{90} = frac{329}{90} = 3 + frac{59}{90} $$
            And now we take our bounds of $a$ to take on the worst-case-scenario for the sinusoidal part of the expression
            $$2sin(frac{4}{11}-1)+sin(1) > -frac{10}{11}+1-frac{1}{6} > -frac{12-11-2}{12}=-frac{1}{12}$$
            And now we have proved the inequality around the boundary regions, since $frac{59}{90}>frac{1}{12}$



            Now that that's done, in a similar vein we can show this for our first interval $2<c leq 3$. Firstly, we assume WLOG that $c geq a,b$ and now we can establish new bounds for $a$ and $b$ by our constraint $b = frac{a+c}{3ca-1}$ to get $frac{2c}{3c^2-1} leq a,b leq c$. Note that this minimum is strictly decreasing for $c>2$ since the denominator increases much faster than the numerator, so the lowest point we find is at $c=3$, and that is $frac{3}{13}$. Now let's again find the minimum of our expression $a+b$ using the derivative again since
            $$D_a left(a+frac{1}{3}+frac{c+frac{1}{3c}}{3ca-1}right)=1-frac{3c^2+1}{(3ca-1)^2}$$
            And setting it equal to $0$ again we get the solution $a=b=frac{1+sqrt{1+3c^2}}{3c}$. This is again the minimum since because $3ca-1$ is increasing, $D_a$ will be positive for $a>frac{1+sqrt{1+3c^2}}{3c}$, and the function is still symmetric. The minimum of $ab+c(a+b)=(c+frac{1}{3c})(a+b)$ is thus
            $$s(c) = left(c+frac{1}{3c}right)left(2frac{frac{1}{c}+sqrt{frac{1}{c^2}+3}}{3}right)+frac{1}{3}$$
            Note that this $s(x)$ is increasing for $c>2$ since
            $$D_c (s(c)) = frac{2}{3}left(1-frac{1}{3c^2}right)left(frac{1}{c} + frac{2}{3}sqrt{frac{1}{c^2}+3}right) - left(c+frac{1}{3c}right)left(frac{1}{c^2}+frac{1}{c^3} frac{1}{sqrt{frac{1}{c^2}+3}}right) > 0 $$
            $$iff left(c^2-frac{1}{3}right)left(1+sqrt{1+3c^2}right)-left(c^2+frac{1}{3}right)left(1+frac{1}{sqrt{1+3c^2}}right)>0$$
            $$iff -frac{2}{3}+c^2left(sqrt{1+3c^2}-frac{1}{sqrt{1+3c^2}}right)-frac{1}{3}left(sqrt{1+3c^2}+frac{1}{sqrt{1+3c^2}}right) > 0$$
            And since $c>2$, this is implied by
            $$-frac{2}{3}+c^2left(sqrt{1+3c^2}-frac{1}{3}right)-frac{1}{3}left(sqrt{1+3c^2}+frac{1}{3}right) > 0$$
            $$iff -frac{8}{9}+left(c^2-frac{1}{3}right)left(sqrt{1+3c^2}-frac{1}{3}right) > 0$$
            And now, since we finally have two increasing functions multiplied together, we can replace each with their lowest value to prove the inequality. Thus since
            $$left(4-frac{1}{3}right)left(sqrt{13}-frac{1}{3}right)-frac{8}{9}>0$$
            $s(c)$ is increasing.



            Now we have shown that $s(c)$ is increasing and that the minimum of $a,b$ and thus the minimum of $sin(a-1)$ since $0<a<3$. To prove this inequality for $2<c<3$, all that is left is to do is plug in our worst case scenario. So we have to show
            $$s(2)-2sin(frac{3}{13}-1)+sin(1)>3$$
            Since $sin(1)$ is the minimum of $sin(c-1)$ over $2<c leq 3$. We already calculated $s(2)$ so now we use a few taylor expansions to obtain that this inequality is implied by
            $$3+frac{59}{90}-2left(frac{10}{13}-frac{left(frac{10}{13}right)^3}{3!}+frac{left(frac{10}{13}right)^5}{5!}right)+1-frac{1}{6} > 3$$
            Which you can do out yourself if you'd like. At this point I will stop doing out these calculations as they make the answer tedious and long.



            For our next interval $3<c<1 + pi$ we can use a better bound for the minimum and maximum of $a,b$ since if $ab+c(a+b)>5$ then it follows that $$ab+c(a+b) + sin(a-1)+ sin(b-1) + sin(c-1) > 5 - 1 - 1 + 0 = 3$$. So all cases where $ab+c(a+b)>5$ are accounted for and this imposing at least the maximum
            $a,b<a+b<frac{5}{c}$, and this implies the minimum $a,b>frac{2(5/c)}{3c(5/c)-1}=frac{5}{7c}$. Note that this function is still a decreasing one and now we have over the interval $3<c<1+pi<frac{7+22}{7}$ that $a,b > frac{5}{29}$. Using this, we can now prove our inequality by showing
            $$3 < ab+c(a+b) + sin(a-1) + sin(b-1) + sin(c-1) < s(3) + 2sin(5/29-1) < s(3) -48/29 approx 3.02784 $$
            And this can be trivially shown by bounding $s(3)$ as well.



            Next interval: $1+pi < c < 1 + pi + 1$. We can no longer use the bound from the previous problem, but we can at this point just say
            $$s(1+pi) + sin(a-1)+sin(b-1)+sin(c-1) > s(4) -3sin(1) approx 3.25 > 3$$
            This can again easily be done using a quick taylor expansion, and that $s(4)=frac{52}{9}$



            Finally for our interval $c>2+pi$ we finally have $s(2+pi)>6$, so we are done.



            Note: There is probably at least one mistake in here, given the length, so feel free to leave comments.






            share|cite|improve this answer









            $endgroup$



            Alright, I think I've finally got a solution, but be warned, it is not a particularly nice one. First off, let's establish the minimum within $0<a,b,cleq2$, (namely $a=b=c=1$) by using lagrange multipliers, then we'll worry about the case where the numbers can be greater than $2$. So, let's call our constraint function
            $$g(a,b,c)=a+b+c-3abc$$
            $$nabla g(a,b,c) = [1-3bc,1-3ca,1-3ab]$$
            And our function to be minimized
            $$f(a,b,c)=ab+bc+ca+sin(a-1)+sin(b-1)+sin(c-1)$$
            $$nabla f(a,b,c) = [b+c+cos(a-1),c+a+cos(b-1),a+b+cos(c-1)]$$
            Now using lagrange multipliers we can write
            $$lambda (1-3bc)=b+c+cos(a-1)$$
            and so on, but to make things easier and symmetrical, let's make a new variable $k=a+b+c=3abcgeq 3$ so we can write instead
            $$lambda =h(a)=frac{k-a+cos(a-1)}{1-frac{k}{a}}=-a+frac{cos(a-1)}{1-frac{k}{a}}$$
            Note that we can write $lambda$ in this way replacing $a$ with $b$ or $c$ as it is nice and symmetrical. In order to show that there is one unique solution, I came up with the following argument (which is the only vaguely interesting part of this entire proof, so stop reading after this if you don't want to be bothered with the rest of this mess). If $h(a)$ is a strictly decreasing (or increasing) function over our interval $0<aleq2$, then in order for $lambda$ to be constant, we must have $a=b=c$, and if $a=b=c$, then we have $a+a+a=3aaa$ with the only positive solution being $a=b=c=1$ giving $f(a,b,c)=3$ as our local minimum.



            MESSY PART Now to show that $h(x)$ is in fact strictly decreasing over our interval $(0,2]$, so let's take the derivative! We want
            $$0>h'(x)=-1+frac{-(1-frac{k}{a})sin(a-1)-cos(a-1)frac{k}{a^2}}{(1-frac{k}{a})^2}$$
            This inequality is implied by
            $$0>(ka-a^2)sin(a-1) - kcos(a-1) - (k-a)^2$$
            which is in turn implied by
            $$0>(k-a)asin(1)-kcos(1) - (k-a)^2$$
            since $sin(1)$ is the maximum of $sin(a-1)$ and $cos(1)$ is the maximum of $-cos(a-1)$ over the interval $a in (0,2]$
            Now this expression is strictly increasing over the interval since
            $$0<(k-2a)sin(1)-2a+2k$$
            because
            $$0<-1-4+6 < (3-4)sin(1)-4+6$$



            So our maximum lies at $a=2$, and when we check that against our previous inequality, we get
            $$0>(2k-4)sin(1)-kcos(1) - (k-2)^2$$
            And this function is strictly decreasing for $k>3$ since
            $$0geq 2+4-2 cdot 3 - 0 > 2sin(1)+4-2k - cos(1)$$
            and if we plug in $k=3$ we get a true inequality, that is
            $$0>(2 cdot 3-4)sin(1)-3cos(1)- (3-2)^2$$
            This can be easily proved by bounding $sin(1)$ and $cos(1)$ with a short Taylor expansion, namely $sin(1)=1$ (ending on a positive for the upper bound) and $cos(1)=1-frac{1}{2}$ (ending on a negative for the lower bound). So we get
            $$0>2 - 3 frac{1}{2} - 1$$
            Thus our original inequality holds and the unique extremum is $a=b=c=1$ for $0<a,b,c<2$



            And now we must check the boundary cases. Note that this function and constraint is completely symmetrical so we only need to check where $c=2$. Rewriting the inequality we now must prove
            $$ab+2(a+b)+sin(a-1)+sin(b-1)+sin(1)>3$$
            with the constraints $a+b+2=6ab$ and $a,b leq 2$



            We can rewrite the first constraint as $b = frac{a+2}{6a-1} = 1/6 + frac{2+1/6}{6a-1}$ and we now we can refine the constraint on $a$ and $b$ since we know that this function of $a$ is decreasing for $a>1/6$ so we know the lowest value of $b$ (and thus $a$) is when $a=2$, so we also have $a,b geq frac{4}{11}$
            Now the minimum of $ab+2(a+b)=frac{1}{3}+frac{13}{6}(a+frac{1}{6} + frac{13/6}{6a-1})$ and thus really just $a+b$ can be found by taking the derivative yet again. Doing so we get
            $$D_a(a+frac{1}{3}+frac{13/6}{6a-1})=1-frac{13}{(6a-1)^2}$$
            And setting it equal to $0$ we get the solution $a=b=frac{1+sqrt{13}}{6}$. Now for $a>frac{1+sqrt{13}}{6}$, since $6a-1$ is clearly increasing, $D_a$ will be postive, and since this is symmetric, this means that $a=b=frac{1+sqrt{13}}{6}$ must be the minimum.



            Now that we have this, let's plug it in! Doing so we get our minimum to be
            $$(2+frac{1}{6})(2frac{1+sqrt{13}}{6})+frac{1}{3}>frac{13}{6} cdot frac{1+3.6}{3}+frac{1}{3}=frac{13 cdot 23 + 30}{90} = frac{329}{90} = 3 + frac{59}{90} $$
            And now we take our bounds of $a$ to take on the worst-case-scenario for the sinusoidal part of the expression
            $$2sin(frac{4}{11}-1)+sin(1) > -frac{10}{11}+1-frac{1}{6} > -frac{12-11-2}{12}=-frac{1}{12}$$
            And now we have proved the inequality around the boundary regions, since $frac{59}{90}>frac{1}{12}$



            Now that that's done, in a similar vein we can show this for our first interval $2<c leq 3$. Firstly, we assume WLOG that $c geq a,b$ and now we can establish new bounds for $a$ and $b$ by our constraint $b = frac{a+c}{3ca-1}$ to get $frac{2c}{3c^2-1} leq a,b leq c$. Note that this minimum is strictly decreasing for $c>2$ since the denominator increases much faster than the numerator, so the lowest point we find is at $c=3$, and that is $frac{3}{13}$. Now let's again find the minimum of our expression $a+b$ using the derivative again since
            $$D_a left(a+frac{1}{3}+frac{c+frac{1}{3c}}{3ca-1}right)=1-frac{3c^2+1}{(3ca-1)^2}$$
            And setting it equal to $0$ again we get the solution $a=b=frac{1+sqrt{1+3c^2}}{3c}$. This is again the minimum since because $3ca-1$ is increasing, $D_a$ will be positive for $a>frac{1+sqrt{1+3c^2}}{3c}$, and the function is still symmetric. The minimum of $ab+c(a+b)=(c+frac{1}{3c})(a+b)$ is thus
            $$s(c) = left(c+frac{1}{3c}right)left(2frac{frac{1}{c}+sqrt{frac{1}{c^2}+3}}{3}right)+frac{1}{3}$$
            Note that this $s(x)$ is increasing for $c>2$ since
            $$D_c (s(c)) = frac{2}{3}left(1-frac{1}{3c^2}right)left(frac{1}{c} + frac{2}{3}sqrt{frac{1}{c^2}+3}right) - left(c+frac{1}{3c}right)left(frac{1}{c^2}+frac{1}{c^3} frac{1}{sqrt{frac{1}{c^2}+3}}right) > 0 $$
            $$iff left(c^2-frac{1}{3}right)left(1+sqrt{1+3c^2}right)-left(c^2+frac{1}{3}right)left(1+frac{1}{sqrt{1+3c^2}}right)>0$$
            $$iff -frac{2}{3}+c^2left(sqrt{1+3c^2}-frac{1}{sqrt{1+3c^2}}right)-frac{1}{3}left(sqrt{1+3c^2}+frac{1}{sqrt{1+3c^2}}right) > 0$$
            And since $c>2$, this is implied by
            $$-frac{2}{3}+c^2left(sqrt{1+3c^2}-frac{1}{3}right)-frac{1}{3}left(sqrt{1+3c^2}+frac{1}{3}right) > 0$$
            $$iff -frac{8}{9}+left(c^2-frac{1}{3}right)left(sqrt{1+3c^2}-frac{1}{3}right) > 0$$
            And now, since we finally have two increasing functions multiplied together, we can replace each with their lowest value to prove the inequality. Thus since
            $$left(4-frac{1}{3}right)left(sqrt{13}-frac{1}{3}right)-frac{8}{9}>0$$
            $s(c)$ is increasing.



            Now we have shown that $s(c)$ is increasing and that the minimum of $a,b$ and thus the minimum of $sin(a-1)$ since $0<a<3$. To prove this inequality for $2<c<3$, all that is left is to do is plug in our worst case scenario. So we have to show
            $$s(2)-2sin(frac{3}{13}-1)+sin(1)>3$$
            Since $sin(1)$ is the minimum of $sin(c-1)$ over $2<c leq 3$. We already calculated $s(2)$ so now we use a few taylor expansions to obtain that this inequality is implied by
            $$3+frac{59}{90}-2left(frac{10}{13}-frac{left(frac{10}{13}right)^3}{3!}+frac{left(frac{10}{13}right)^5}{5!}right)+1-frac{1}{6} > 3$$
            Which you can do out yourself if you'd like. At this point I will stop doing out these calculations as they make the answer tedious and long.



            For our next interval $3<c<1 + pi$ we can use a better bound for the minimum and maximum of $a,b$ since if $ab+c(a+b)>5$ then it follows that $$ab+c(a+b) + sin(a-1)+ sin(b-1) + sin(c-1) > 5 - 1 - 1 + 0 = 3$$. So all cases where $ab+c(a+b)>5$ are accounted for and this imposing at least the maximum
            $a,b<a+b<frac{5}{c}$, and this implies the minimum $a,b>frac{2(5/c)}{3c(5/c)-1}=frac{5}{7c}$. Note that this function is still a decreasing one and now we have over the interval $3<c<1+pi<frac{7+22}{7}$ that $a,b > frac{5}{29}$. Using this, we can now prove our inequality by showing
            $$3 < ab+c(a+b) + sin(a-1) + sin(b-1) + sin(c-1) < s(3) + 2sin(5/29-1) < s(3) -48/29 approx 3.02784 $$
            And this can be trivially shown by bounding $s(3)$ as well.



            Next interval: $1+pi < c < 1 + pi + 1$. We can no longer use the bound from the previous problem, but we can at this point just say
            $$s(1+pi) + sin(a-1)+sin(b-1)+sin(c-1) > s(4) -3sin(1) approx 3.25 > 3$$
            This can again easily be done using a quick taylor expansion, and that $s(4)=frac{52}{9}$



            Finally for our interval $c>2+pi$ we finally have $s(2+pi)>6$, so we are done.



            Note: There is probably at least one mistake in here, given the length, so feel free to leave comments.







            share|cite|improve this answer












            share|cite|improve this answer



            share|cite|improve this answer










            answered Jul 9 '18 at 22:06









            Isaac BrowneIsaac Browne

            4,59731132




            4,59731132























                1












                $begingroup$



                $$mathbf{color{brown}{Analysis of the issue condition.}}$$



                Let WLOG
                $$0<ale ble c,tag1$$
                $$a+b+c=3abc,tag2$$
                then
                $$c+a=(3ac-1)b,quad (3ac-1)ale a+cle(3ac-1)c,$$
                $$begin{cases}
                (3a^2-1)cle2a\[4pt]
                (3c^2-1)ale2c\[4pt]
                0<ale ble c,
                end{cases}rightarrow
                begin{cases}
                dfrac1{sqrt3} < ale dfrac{2c}{3c^2-1}\
                ale cledfrac{2a}{3a^2-1}\
                dfrac{2c}{3c^2-1}le ale c\
                end{cases}rightarrow
                begin{cases}
                dfrac1{sqrt3}< a\[4pt]
                (sqrt3c-1)^2le2\[4pt]
                3a^2-1le 2le 3c^2-1\[4pt]
                end{cases}$$
                $$boxed {dfrac1{sqrt3}<ale1le cledfrac{sqrt2+1}{sqrt3},quad b=dfrac{a+c}{3ac-1}.}tag3$$
                Besides, as shown in OP,
                $$dfrac13left(dfrac1{ab}+dfrac1{bc}+dfrac1{ca}right)=1.$$
                Using AM-HM, easy to obtain
                $$ab+bc+acge3.tag4$$





                $$mathbf{color{brown}{The trig inequality}}$$



                Let us consider the inequality
                $$sin(a-1) + sin(b-1) + sin(c-1) +lambda(a+b+c -3abc)tag5$$
                under the constraints $(3).$



                Using Lagrange multipiers method with the function
                $$f(a,b,c,lambda) = sin(a-1) + sin(b-1) + sin(c-1) +lambda(a+b+c -3abc),$$
                the stationary points can be found from the system $f'_a=f'_b=f'_c=f'_lambda=0,$ or
                begin{cases}
                cos(a-1)+ lambda(1-3bc)=0\
                cos(b-1)+ lambda(1-3ca)=0\
                cos(c-1)+ lambda(1-3ab)=0\
                a+b+c=3abc\
                dfrac{sqrt3}3<ale1le cledfrac{sqrt2+1}{sqrt3},\
                end{cases}
                begin{cases}
                dfrac1{3bc-1}cos(a-1) - lambda = 0\
                dfrac1{3ca-1}cos(b-1) - lambda = 0\
                dfrac1{3ab-1}cos(c-1) - lambda = 0\
                a+b+c=3abc\
                dfrac{sqrt3}3<ale1le cledfrac{sqrt2+1}{sqrt3},\
                end{cases}
                begin{cases}
                b =dfrac{a+c}{3ac-1}\[4pt]
                3bc-1=dfrac{3ac+3c^2}{3ac-1}-1 = dfrac{3c^2+1}{3ac-1}\[4pt]
                3ab-1=dfrac{3a^2+3ac}{3ac-1}-1 = dfrac{3a^2+1}{3ac-1}\[4pt]
                dfrac{3ac-1}{3c^2+1}cos(a-1) = dfrac{3ac-1}{3a^2+1}cos(c-1)\[4pt]
                dfrac{3ac-1}{3c^2+1}cos(a-1) = dfrac1{3ac-1}cosleft(dfrac{a+c}{3ac-1}-1right)\[4pt]
                dfrac{sqrt3}3<ale1le cledfrac{sqrt2+1}{sqrt3}.tag6
                end{cases}
                Function $g(x)=(3x^2+1)cos (x-1)$ has positive derivation
                $$g'(x)=6xcos(x-1)-(3x^2+1)sin(x-1)$$
                in $left[dfrac1{sqrt3},dfrac{sqrt2+1}{sqrt3}right],$ so $g(x)$ increases in this interval.



                The equation $(6.4)$ can be rewrited as
                $$g(a)=g(c).$$
                Taking in account constraints $(6.6),$ the system $(6)$ can be satisfied in the single point $$a=1,quad c=g(1)=1.$$
                This gives the single stationary point
                $$a=b=c=1,quad f(a,b,c,lambda)=0.$$



                Note that constraints $(6.6)$ determines the blurring of boundaries between variables and can not be checked.
                On the other hand, checking of the trig inequality in the random point $$left(a,b,c, fright)=left(dfrac23,dfrac65,dfrac43, sindfrac15right)$$ shows that trig inequality holds. Therefore, it is proved.



                Taking in account $(4)$, the issue inequality is proved.






                share|cite|improve this answer











                $endgroup$


















                  1












                  $begingroup$



                  $$mathbf{color{brown}{Analysis of the issue condition.}}$$



                  Let WLOG
                  $$0<ale ble c,tag1$$
                  $$a+b+c=3abc,tag2$$
                  then
                  $$c+a=(3ac-1)b,quad (3ac-1)ale a+cle(3ac-1)c,$$
                  $$begin{cases}
                  (3a^2-1)cle2a\[4pt]
                  (3c^2-1)ale2c\[4pt]
                  0<ale ble c,
                  end{cases}rightarrow
                  begin{cases}
                  dfrac1{sqrt3} < ale dfrac{2c}{3c^2-1}\
                  ale cledfrac{2a}{3a^2-1}\
                  dfrac{2c}{3c^2-1}le ale c\
                  end{cases}rightarrow
                  begin{cases}
                  dfrac1{sqrt3}< a\[4pt]
                  (sqrt3c-1)^2le2\[4pt]
                  3a^2-1le 2le 3c^2-1\[4pt]
                  end{cases}$$
                  $$boxed {dfrac1{sqrt3}<ale1le cledfrac{sqrt2+1}{sqrt3},quad b=dfrac{a+c}{3ac-1}.}tag3$$
                  Besides, as shown in OP,
                  $$dfrac13left(dfrac1{ab}+dfrac1{bc}+dfrac1{ca}right)=1.$$
                  Using AM-HM, easy to obtain
                  $$ab+bc+acge3.tag4$$





                  $$mathbf{color{brown}{The trig inequality}}$$



                  Let us consider the inequality
                  $$sin(a-1) + sin(b-1) + sin(c-1) +lambda(a+b+c -3abc)tag5$$
                  under the constraints $(3).$



                  Using Lagrange multipiers method with the function
                  $$f(a,b,c,lambda) = sin(a-1) + sin(b-1) + sin(c-1) +lambda(a+b+c -3abc),$$
                  the stationary points can be found from the system $f'_a=f'_b=f'_c=f'_lambda=0,$ or
                  begin{cases}
                  cos(a-1)+ lambda(1-3bc)=0\
                  cos(b-1)+ lambda(1-3ca)=0\
                  cos(c-1)+ lambda(1-3ab)=0\
                  a+b+c=3abc\
                  dfrac{sqrt3}3<ale1le cledfrac{sqrt2+1}{sqrt3},\
                  end{cases}
                  begin{cases}
                  dfrac1{3bc-1}cos(a-1) - lambda = 0\
                  dfrac1{3ca-1}cos(b-1) - lambda = 0\
                  dfrac1{3ab-1}cos(c-1) - lambda = 0\
                  a+b+c=3abc\
                  dfrac{sqrt3}3<ale1le cledfrac{sqrt2+1}{sqrt3},\
                  end{cases}
                  begin{cases}
                  b =dfrac{a+c}{3ac-1}\[4pt]
                  3bc-1=dfrac{3ac+3c^2}{3ac-1}-1 = dfrac{3c^2+1}{3ac-1}\[4pt]
                  3ab-1=dfrac{3a^2+3ac}{3ac-1}-1 = dfrac{3a^2+1}{3ac-1}\[4pt]
                  dfrac{3ac-1}{3c^2+1}cos(a-1) = dfrac{3ac-1}{3a^2+1}cos(c-1)\[4pt]
                  dfrac{3ac-1}{3c^2+1}cos(a-1) = dfrac1{3ac-1}cosleft(dfrac{a+c}{3ac-1}-1right)\[4pt]
                  dfrac{sqrt3}3<ale1le cledfrac{sqrt2+1}{sqrt3}.tag6
                  end{cases}
                  Function $g(x)=(3x^2+1)cos (x-1)$ has positive derivation
                  $$g'(x)=6xcos(x-1)-(3x^2+1)sin(x-1)$$
                  in $left[dfrac1{sqrt3},dfrac{sqrt2+1}{sqrt3}right],$ so $g(x)$ increases in this interval.



                  The equation $(6.4)$ can be rewrited as
                  $$g(a)=g(c).$$
                  Taking in account constraints $(6.6),$ the system $(6)$ can be satisfied in the single point $$a=1,quad c=g(1)=1.$$
                  This gives the single stationary point
                  $$a=b=c=1,quad f(a,b,c,lambda)=0.$$



                  Note that constraints $(6.6)$ determines the blurring of boundaries between variables and can not be checked.
                  On the other hand, checking of the trig inequality in the random point $$left(a,b,c, fright)=left(dfrac23,dfrac65,dfrac43, sindfrac15right)$$ shows that trig inequality holds. Therefore, it is proved.



                  Taking in account $(4)$, the issue inequality is proved.






                  share|cite|improve this answer











                  $endgroup$
















                    1












                    1








                    1





                    $begingroup$



                    $$mathbf{color{brown}{Analysis of the issue condition.}}$$



                    Let WLOG
                    $$0<ale ble c,tag1$$
                    $$a+b+c=3abc,tag2$$
                    then
                    $$c+a=(3ac-1)b,quad (3ac-1)ale a+cle(3ac-1)c,$$
                    $$begin{cases}
                    (3a^2-1)cle2a\[4pt]
                    (3c^2-1)ale2c\[4pt]
                    0<ale ble c,
                    end{cases}rightarrow
                    begin{cases}
                    dfrac1{sqrt3} < ale dfrac{2c}{3c^2-1}\
                    ale cledfrac{2a}{3a^2-1}\
                    dfrac{2c}{3c^2-1}le ale c\
                    end{cases}rightarrow
                    begin{cases}
                    dfrac1{sqrt3}< a\[4pt]
                    (sqrt3c-1)^2le2\[4pt]
                    3a^2-1le 2le 3c^2-1\[4pt]
                    end{cases}$$
                    $$boxed {dfrac1{sqrt3}<ale1le cledfrac{sqrt2+1}{sqrt3},quad b=dfrac{a+c}{3ac-1}.}tag3$$
                    Besides, as shown in OP,
                    $$dfrac13left(dfrac1{ab}+dfrac1{bc}+dfrac1{ca}right)=1.$$
                    Using AM-HM, easy to obtain
                    $$ab+bc+acge3.tag4$$





                    $$mathbf{color{brown}{The trig inequality}}$$



                    Let us consider the inequality
                    $$sin(a-1) + sin(b-1) + sin(c-1) +lambda(a+b+c -3abc)tag5$$
                    under the constraints $(3).$



                    Using Lagrange multipiers method with the function
                    $$f(a,b,c,lambda) = sin(a-1) + sin(b-1) + sin(c-1) +lambda(a+b+c -3abc),$$
                    the stationary points can be found from the system $f'_a=f'_b=f'_c=f'_lambda=0,$ or
                    begin{cases}
                    cos(a-1)+ lambda(1-3bc)=0\
                    cos(b-1)+ lambda(1-3ca)=0\
                    cos(c-1)+ lambda(1-3ab)=0\
                    a+b+c=3abc\
                    dfrac{sqrt3}3<ale1le cledfrac{sqrt2+1}{sqrt3},\
                    end{cases}
                    begin{cases}
                    dfrac1{3bc-1}cos(a-1) - lambda = 0\
                    dfrac1{3ca-1}cos(b-1) - lambda = 0\
                    dfrac1{3ab-1}cos(c-1) - lambda = 0\
                    a+b+c=3abc\
                    dfrac{sqrt3}3<ale1le cledfrac{sqrt2+1}{sqrt3},\
                    end{cases}
                    begin{cases}
                    b =dfrac{a+c}{3ac-1}\[4pt]
                    3bc-1=dfrac{3ac+3c^2}{3ac-1}-1 = dfrac{3c^2+1}{3ac-1}\[4pt]
                    3ab-1=dfrac{3a^2+3ac}{3ac-1}-1 = dfrac{3a^2+1}{3ac-1}\[4pt]
                    dfrac{3ac-1}{3c^2+1}cos(a-1) = dfrac{3ac-1}{3a^2+1}cos(c-1)\[4pt]
                    dfrac{3ac-1}{3c^2+1}cos(a-1) = dfrac1{3ac-1}cosleft(dfrac{a+c}{3ac-1}-1right)\[4pt]
                    dfrac{sqrt3}3<ale1le cledfrac{sqrt2+1}{sqrt3}.tag6
                    end{cases}
                    Function $g(x)=(3x^2+1)cos (x-1)$ has positive derivation
                    $$g'(x)=6xcos(x-1)-(3x^2+1)sin(x-1)$$
                    in $left[dfrac1{sqrt3},dfrac{sqrt2+1}{sqrt3}right],$ so $g(x)$ increases in this interval.



                    The equation $(6.4)$ can be rewrited as
                    $$g(a)=g(c).$$
                    Taking in account constraints $(6.6),$ the system $(6)$ can be satisfied in the single point $$a=1,quad c=g(1)=1.$$
                    This gives the single stationary point
                    $$a=b=c=1,quad f(a,b,c,lambda)=0.$$



                    Note that constraints $(6.6)$ determines the blurring of boundaries between variables and can not be checked.
                    On the other hand, checking of the trig inequality in the random point $$left(a,b,c, fright)=left(dfrac23,dfrac65,dfrac43, sindfrac15right)$$ shows that trig inequality holds. Therefore, it is proved.



                    Taking in account $(4)$, the issue inequality is proved.






                    share|cite|improve this answer











                    $endgroup$





                    $$mathbf{color{brown}{Analysis of the issue condition.}}$$



                    Let WLOG
                    $$0<ale ble c,tag1$$
                    $$a+b+c=3abc,tag2$$
                    then
                    $$c+a=(3ac-1)b,quad (3ac-1)ale a+cle(3ac-1)c,$$
                    $$begin{cases}
                    (3a^2-1)cle2a\[4pt]
                    (3c^2-1)ale2c\[4pt]
                    0<ale ble c,
                    end{cases}rightarrow
                    begin{cases}
                    dfrac1{sqrt3} < ale dfrac{2c}{3c^2-1}\
                    ale cledfrac{2a}{3a^2-1}\
                    dfrac{2c}{3c^2-1}le ale c\
                    end{cases}rightarrow
                    begin{cases}
                    dfrac1{sqrt3}< a\[4pt]
                    (sqrt3c-1)^2le2\[4pt]
                    3a^2-1le 2le 3c^2-1\[4pt]
                    end{cases}$$
                    $$boxed {dfrac1{sqrt3}<ale1le cledfrac{sqrt2+1}{sqrt3},quad b=dfrac{a+c}{3ac-1}.}tag3$$
                    Besides, as shown in OP,
                    $$dfrac13left(dfrac1{ab}+dfrac1{bc}+dfrac1{ca}right)=1.$$
                    Using AM-HM, easy to obtain
                    $$ab+bc+acge3.tag4$$





                    $$mathbf{color{brown}{The trig inequality}}$$



                    Let us consider the inequality
                    $$sin(a-1) + sin(b-1) + sin(c-1) +lambda(a+b+c -3abc)tag5$$
                    under the constraints $(3).$



                    Using Lagrange multipiers method with the function
                    $$f(a,b,c,lambda) = sin(a-1) + sin(b-1) + sin(c-1) +lambda(a+b+c -3abc),$$
                    the stationary points can be found from the system $f'_a=f'_b=f'_c=f'_lambda=0,$ or
                    begin{cases}
                    cos(a-1)+ lambda(1-3bc)=0\
                    cos(b-1)+ lambda(1-3ca)=0\
                    cos(c-1)+ lambda(1-3ab)=0\
                    a+b+c=3abc\
                    dfrac{sqrt3}3<ale1le cledfrac{sqrt2+1}{sqrt3},\
                    end{cases}
                    begin{cases}
                    dfrac1{3bc-1}cos(a-1) - lambda = 0\
                    dfrac1{3ca-1}cos(b-1) - lambda = 0\
                    dfrac1{3ab-1}cos(c-1) - lambda = 0\
                    a+b+c=3abc\
                    dfrac{sqrt3}3<ale1le cledfrac{sqrt2+1}{sqrt3},\
                    end{cases}
                    begin{cases}
                    b =dfrac{a+c}{3ac-1}\[4pt]
                    3bc-1=dfrac{3ac+3c^2}{3ac-1}-1 = dfrac{3c^2+1}{3ac-1}\[4pt]
                    3ab-1=dfrac{3a^2+3ac}{3ac-1}-1 = dfrac{3a^2+1}{3ac-1}\[4pt]
                    dfrac{3ac-1}{3c^2+1}cos(a-1) = dfrac{3ac-1}{3a^2+1}cos(c-1)\[4pt]
                    dfrac{3ac-1}{3c^2+1}cos(a-1) = dfrac1{3ac-1}cosleft(dfrac{a+c}{3ac-1}-1right)\[4pt]
                    dfrac{sqrt3}3<ale1le cledfrac{sqrt2+1}{sqrt3}.tag6
                    end{cases}
                    Function $g(x)=(3x^2+1)cos (x-1)$ has positive derivation
                    $$g'(x)=6xcos(x-1)-(3x^2+1)sin(x-1)$$
                    in $left[dfrac1{sqrt3},dfrac{sqrt2+1}{sqrt3}right],$ so $g(x)$ increases in this interval.



                    The equation $(6.4)$ can be rewrited as
                    $$g(a)=g(c).$$
                    Taking in account constraints $(6.6),$ the system $(6)$ can be satisfied in the single point $$a=1,quad c=g(1)=1.$$
                    This gives the single stationary point
                    $$a=b=c=1,quad f(a,b,c,lambda)=0.$$



                    Note that constraints $(6.6)$ determines the blurring of boundaries between variables and can not be checked.
                    On the other hand, checking of the trig inequality in the random point $$left(a,b,c, fright)=left(dfrac23,dfrac65,dfrac43, sindfrac15right)$$ shows that trig inequality holds. Therefore, it is proved.



                    Taking in account $(4)$, the issue inequality is proved.







                    share|cite|improve this answer














                    share|cite|improve this answer



                    share|cite|improve this answer








                    edited Jul 10 '18 at 7:46

























                    answered Jul 9 '18 at 23:15









                    Yuri NegometyanovYuri Negometyanov

                    11.2k1728




                    11.2k1728























                        0












                        $begingroup$

                        Hint : Prove with Jensen's inequality (apply to the function $f(x)=0.5x^2+sin(x-1)$ ) this with the obvious restriction $ab+bc+caleq6$:




                        $$ab+bc+ca+sin(a-1)+sin(b-1)+sin(c-1)$$$$geq$$$$ ab+bc+ca-0.5(a^2+b^2+c^2)+3sin(frac{a+b+c}{3}-1)+1.5(frac{a+b+c}{3})^2$$




                        And now a last hint prove this (with the condition $3abc=a+b+c$ and $ab+bc+caleq6$ ):




                        $$ab+bc+ca-0.5(a^2+b^2+c^2)+1.5(frac{a+b+c}{3})^2geq 3$$
                        And
                        $$3sin(frac{a+b+c}{3}-1)geq 0$$







                        share|cite|improve this answer









                        $endgroup$


















                          0












                          $begingroup$

                          Hint : Prove with Jensen's inequality (apply to the function $f(x)=0.5x^2+sin(x-1)$ ) this with the obvious restriction $ab+bc+caleq6$:




                          $$ab+bc+ca+sin(a-1)+sin(b-1)+sin(c-1)$$$$geq$$$$ ab+bc+ca-0.5(a^2+b^2+c^2)+3sin(frac{a+b+c}{3}-1)+1.5(frac{a+b+c}{3})^2$$




                          And now a last hint prove this (with the condition $3abc=a+b+c$ and $ab+bc+caleq6$ ):




                          $$ab+bc+ca-0.5(a^2+b^2+c^2)+1.5(frac{a+b+c}{3})^2geq 3$$
                          And
                          $$3sin(frac{a+b+c}{3}-1)geq 0$$







                          share|cite|improve this answer









                          $endgroup$
















                            0












                            0








                            0





                            $begingroup$

                            Hint : Prove with Jensen's inequality (apply to the function $f(x)=0.5x^2+sin(x-1)$ ) this with the obvious restriction $ab+bc+caleq6$:




                            $$ab+bc+ca+sin(a-1)+sin(b-1)+sin(c-1)$$$$geq$$$$ ab+bc+ca-0.5(a^2+b^2+c^2)+3sin(frac{a+b+c}{3}-1)+1.5(frac{a+b+c}{3})^2$$




                            And now a last hint prove this (with the condition $3abc=a+b+c$ and $ab+bc+caleq6$ ):




                            $$ab+bc+ca-0.5(a^2+b^2+c^2)+1.5(frac{a+b+c}{3})^2geq 3$$
                            And
                            $$3sin(frac{a+b+c}{3}-1)geq 0$$







                            share|cite|improve this answer









                            $endgroup$



                            Hint : Prove with Jensen's inequality (apply to the function $f(x)=0.5x^2+sin(x-1)$ ) this with the obvious restriction $ab+bc+caleq6$:




                            $$ab+bc+ca+sin(a-1)+sin(b-1)+sin(c-1)$$$$geq$$$$ ab+bc+ca-0.5(a^2+b^2+c^2)+3sin(frac{a+b+c}{3}-1)+1.5(frac{a+b+c}{3})^2$$




                            And now a last hint prove this (with the condition $3abc=a+b+c$ and $ab+bc+caleq6$ ):




                            $$ab+bc+ca-0.5(a^2+b^2+c^2)+1.5(frac{a+b+c}{3})^2geq 3$$
                            And
                            $$3sin(frac{a+b+c}{3}-1)geq 0$$








                            share|cite|improve this answer












                            share|cite|improve this answer



                            share|cite|improve this answer










                            answered Dec 7 '18 at 13:23









                            max8128max8128

                            372421




                            372421






























                                draft saved

                                draft discarded




















































                                Thanks for contributing an answer to Mathematics Stack Exchange!


                                • Please be sure to answer the question. Provide details and share your research!

                                But avoid



                                • Asking for help, clarification, or responding to other answers.

                                • Making statements based on opinion; back them up with references or personal experience.


                                Use MathJax to format equations. MathJax reference.


                                To learn more, see our tips on writing great answers.




                                draft saved


                                draft discarded














                                StackExchange.ready(
                                function () {
                                StackExchange.openid.initPostLogin('.new-post-login', 'https%3a%2f%2fmath.stackexchange.com%2fquestions%2f2838516%2fshow-this-inequality-abbcac-sina-1-sinb-1-sinc-1-ge-3%23new-answer', 'question_page');
                                }
                                );

                                Post as a guest















                                Required, but never shown





















































                                Required, but never shown














                                Required, but never shown












                                Required, but never shown







                                Required, but never shown

































                                Required, but never shown














                                Required, but never shown












                                Required, but never shown







                                Required, but never shown







                                Popular posts from this blog

                                How do I know what Microsoft account the skydrive app is syncing to?

                                When does type information flow backwards in C++?

                                Grease: Live!